Need help on polynomial expressions

Need Help On Polynomial Expressions

Answers

Answer 1

Answer:- 10[tex]m^{2}[/tex] + 3m -9

Step-by-step explanation: Given ;  

  A= -3 -m

  B= 3m -5[tex]m^{2}[/tex]

 2B + 3A

        solution

     2B + 3A

substitute A and B in the formula

    2(3m - 5[tex]m^{2}[/tex]) + 3(-3 -m)

    6m - 10[tex]m^{2}[/tex] - 9 - 3m   group like terms

    - 10[tex]m^{2}[/tex] + (6m -3m) -9

     - 10[tex]m^{2}[/tex] + 3m -9


Related Questions

A bus averages 2 miles per hour faster than a motorcycle. If the bus travels 165 miles in the same time it takes the motorcycle to travel 155 miles, then what is the speed of each?

Answers

Answer:

The bus travels at 33 miles per hour while the motorcycle travels at 31 miles per hour

Step-by-step explanation:

Represent the bus average speed with x and the motorcycle average speed with y

Given

[tex]x = y + 2[/tex]

Distance covered by bus = 165 miles

Distance covered by motorcycle in same time = 155 miles

Required

Determine the speed of each

Average Speed is calculated as;

[tex]Average\ Speed = \frac{Distance}{Time}[/tex]

Since the two are measured with the same time, represent time with T

For the bus

[tex]Average\ Speed = \frac{Distance}{Time}[/tex] becomes

[tex]x = \frac{165}{T}[/tex]

Make T the subject of formula

[tex]T = \frac{165}{x}[/tex]

For the motorcycle

[tex]y = \frac{155}{T}[/tex]

Make T the subject of formula

[tex]T = \frac{155}{y}[/tex]

Since, T = T; we have that

[tex]\frac{165}{x} = \frac{155}{y}[/tex]

Cross Multiply

[tex]165y = 155x[/tex]

Substitute [tex]x = y + 2[/tex]

[tex]165y = 155(y+2)[/tex]

Open Bracket

[tex]165y = 155y - 310[/tex]

Collect Like Terms

[tex]165y - 155y = 310[/tex]

[tex]10y = 310[/tex]

Divide both sides by 10

[tex]y = 31[/tex]

Recall that [tex]x = y + 2[/tex]

[tex]x = 31 +2[/tex]

[tex]x = 33[/tex]

Hence;

The bus travels at 33 miles per hour while the motorcycle travels at 31 miles per hour

help with number 1 please. Thank you.​

Answers

Answer:

Hello,

Step-by-step explanation:

Q1:

[tex]\left\{\begin{array}{ccc}x&=&t+\dfrac{1}{t} \\\\y&=&t-\dfrac{1}{t} \\\end{array}\right.\\\\\left\{\begin{array}{ccc}x^2&=&t^2+\dfrac{1}{t^2} +2\\\\y^2&=&t^2+\dfrac{1}{t^2} -2\\\end{array}\right.\\\\\\x^2-y^2=4: \ equilater\ hyperbola.\\[/tex]

Q2:

1)

[tex]\left\{\begin{array}{ccc}x&=&2t^2} \\\\y&=&4t \\\end{array}\right.\\\\\\\left\{\begin{array}{ccc}t&=&\dfrac{y}{4} \\\\x&=&2*(\dfrac{y}{4})^2 \\\end{array}\right.\\\\\\\boxed{x=\dfrac{y^2}{8}} :\ parabola\ with\ x-axis\ as\ axis\ of\ symmetry[/tex]

2)

[tex]y=\dfrac{25}{x} \\[/tex]

equilater hyperbola (centre (0,0))

On the maturity date of a $10,800, 6-month, 8% note, the borrower sends a check that includes the principal and all of the interest due on the note. What is the amount of the borrower's check?​

Answers

Answer: $ 11,232.00

Step-by-step explanation:

Given:  Amount = $ 10,800

Interest Rate = 8%   =0.08

Time in Months= 6.00

Formula :

Interest on Note = (Amount)× (Interest Rate) × ((Time in Months) /12)

= (10800)× (0.08)× (6/12)

= $432

The amount of the borrower's check =(Amount + Interest on Note)

= $ (10,800+432)

= $ 11,232.00

Hence, The amount of the borrower's check = $ 11,232.00

a rectangular plot has 17m length and 14m width. What will be the cost of fencing at a rate of Rs.25 per meter​

Answers

The fence would be the perimeter of the shape.

Perimeter = 17 + 17 + 14 + 14 = 62 meters.

Multiply the price per meter by total meters:

25 x 62 = 1550

For fencing , we have to find the perimeter of rectangle.

[tex] \large\begin{gathered} {\underline{\boxed{ \rm {\red{Formula \: \: Using \: = \: 2 \: ( \: L + B \: )}}}}}\end{gathered}[/tex]

L denotes length of rectangleB denotes breadth of rectangle

L = 17 mB = 14 m

Substuting the values

Perimeter of rectangle = 2 ( L + B )

Perimeter of rectangle = 2 ( 17 m + 14 m )

Perimeter of rectangle = 2 ( 31 m )

Perimeter of rectangle = 62 m.

Hence, the perimeter of rectangle is 62 m.

For Fencing Cost = perimeter × rate of fencing per metre

Fencing Cost = 62 × 25

Fencing cost = 1,550

Hence, the cost of fencing of a rectangular plot is 1,550

i will give brainliest and 5 stars if you help ASAP​

Answers

Answer:

BC = 13.4

Step-by-step explanation:

its a law of cosines S-A-S

a² = b² + c² - 2bc cosA

a² = 12.6² + 4.6² - ( 2 * 12.6 * 4.6 * cos 90 )

a² = 179.92

a = sqrt (179.92)

a = 13.4

Kane's Furniture Store advertised a table at ​9% discount. The original selling price was ​$109​, and the sale price was ​$100. Was the sale price consistent with the​ ad? Explain.

Answers

Answer:

No, the sale price wasn't consistent with the ad

Step-by-step explanation:

The original selling price is 109. 9% of 109 is 9.81. So the sale price should be 99.19.

What is the three hundred and twenty six to the nearest ten

Answers

Hi there!  

»»————- ★ ————-««

I believe your answer is:  

330

»»————- ★ ————-««  

Here’s why:  

⸻⸻⸻⸻

We are rounding the number 326 to the nearest tens.

The '2' is in the tens place, and the '6' is to the right of the digit.

Since the '6' is greater than or equal to 5, then we would round up.

326 ≈ 330

⸻⸻⸻⸻

»»————- ★ ————-««  

Hope this helps you. I apologize if it’s incorrect.  

326 to the nearest 10 would be 330 because 6 rounds up

A bank account earned 3.5% continuously compounded annual interest. After the initial deposit, no deposits or withdrawals were made. At the end of an 8 year period, the balance in the account was $13231.30. What was the dollar amount of the initial deposit? Round your answer to the nearest dollar. Do not include a dollar sign ($) or comma in your answer.

Answers

Answer:

[tex]\large \boxed{\$10000.00}[/tex]

Step-by-step explanation:

We can use the formula for continuously compounded interest.

[tex]\begin{array}{rcl}A & = & Pe^{rt}\\13231.30& = & Pe^{0.035 \times 8}\\& = &Pe^{0.28}\\& = & P\times 1.3231298\\P & = &\dfrac{13231.30}{1.3231298}\\\\&=&\mathbf{10000.00}\\\end{array}\\\text{The initial deposit was $\large \boxed{\mathbf{\$10000.00}}$}[/tex]

At a certain charity fundraiser, some guests will be randomly selected to receive a gift. The probability of receiving a gift is 3/25. Find the odds in favor of receiving a gift. I got 3/28. A child has a box of candies which might have a toy inside. The odds against the box having a toy are 7/2. What is the probability of the box having a toy? I got 7/9. Are my answers correct?

Answers

Answer:

a) Your answer for part a (I got 3/28) is wrong.

Odds are always expressed as ratios after calculating. Therefore, the odds in favour of receiving a gift = 3:22

b) Your answer in part b(I got 7/9) is correct.

The probability of the box having a toy is 7/9.

Step-by-step explanation:

The question above has to do with odds and probability.

It is important to note that odds are expressed in the form of ratios while probabilities are expressed as fractions.

a) At a certain charity fundraiser, some guests will be randomly selected to receive a gift. The probability of receiving a gift is 3/25. Find the odds in favor of receiving a gift.(I got 3/28)

The formula for calculating odds from probability is Odds = Probability / (1 - Probability).

Probability = 3/25

Odds =(3/25)/(1 - 3/25)

= (3/25)/22/25

= 3/25 ÷ 22/25

= 3/25 × 25/22

= 3/22

Note that odds are always expressed as ratios.

Therefore, the odds in favour of receiving a gift = 3:22

Your answer for part a (I got 3/28) is wrong.

The correct answer is 3:22

b) A child has a box of candies which might have a toy inside. The odds against the box having a toy are 7/2. What is the probability of the box having a toy? I got 7/9.

The formula for calculating probability from odds is P = Odds / (Odds + 1).

Odds = 7/2 or 7:2

We convert the odds to fraction when calculating

Probability = Odds / (Odds + 1).

Probability = (7/2)/ (7/2 + 1)

Probability = (7/2)/9/2

Probability = 7/2 ÷ 9/2

= 7/2 × 2/9

= 7/9

Probability is always expressed as a fraction.

Therefore, the probability of the box having a toy is 7/9.

Your answer in part b(I got 7/9) is correct.

ANSWER ASAP CORRECT ANSWER WILL GET BRAINLIEST PLUS 78 POINTS

Answers

9514 1404 393

Answer:

  C)  462

Step-by-step explanation:

The presenters can be chosen 11C6 = 462 different ways.

__

If the teacher were concerned about the order of the presentations, the number of possibilities goes up to 11P6 = 332,640 different ways.

Find X so that m is parallel to n. Identify the postulate or theorem you used. Please help with these 3 problems, I don’t understand it at all

Answers

the corresponding angles should be equal

so, [tex] 5x+15=90 \implies 5x=75\implies x=15^{\circ}[/tex]

will rate you brainliest

Answers

Answer:

A

Step-by-step explanation:

f(x)→g(x)

(0, 0) → (3, -4)

Therefore it increases it x-axis from 0 to 3

And decreases in y-axis from 0 to -4

Use the points to describe the data and determine the line of best fit.
What type of correlation do the data points represent?


What is the equation that best represents the data?

Answers

Answer:

Answer: negative

Y= -500x + 4,500

Step-by-step explanation:

Answer:

C. negative

B. Y= -500x + 4,500

I just did it

1) (1) The selling price (ii) The cost price (iii) Profit The marked price of an article is 15% above its selling price and the cost price is 25% less tha its marked price. Find the discount percent and gain percent.​

Answers

Answer: (iii) Profit The marked price of an article is 15% above its selling price and the cost price is 25% less tha its marked price. Find the discount percent and gain percent.​

Step-by-step explanation:

find the surface area of the prism HURRY

Answers

Answer:

does the answer help you?

M&M plain candies come in various colors. According to the M&M/Mars Department of Consumer Affairs, the distribution of colors for plain M&M candies is as follows. Color Purple Yellow Red Orange Green Blue Brown Percentage 22% 20% 23% 10% 6% 6% 13% Suppose you have a large bag of plain M&M candies and you choose one candy at random. (a) Find P(green candy or blue candy). Are these outcomes mutually exclusive? Why? Yes. Choosing a green and blue M&M is possible. Yes. Choosing a green and blue M&M is not possible. No. Choosing a green and blue M&M is not possible. No. Choosing a green and blue M&M is possible. (b) Find P(yellow candy or red candy). Are these outcomes mutually exclusive? Why? Yes. Choosing a yellow and red M&M is possible. No. Choosing a yellow and red M&M is not possible. Yes. Choosing a yellow and red M&M is not possible. No. Choosing a yellow and red M&M is possible. (c) Find P(not purple candy).

Answers

Answer:

A) 0.12. Yes. Choosing a green and blue M&M is possible

B) 0.43. Yes. Choosing a yellow and red M&M is possible

C) 0.78

Step-by-step explanation:

First of all, the summation of the distribution of all colours is;

Σ(all colors ) = 22% + 20% + 23% + 10% + 6% + 6% + 13% = 100%, or 1.

Thus;

a) P(green candy or blue candy) is;

P(GREEN ∪ BLUE) = P(G) + P(BL)

P(GREEN ∪ BLUE) = 6%+6%

P(GREEN ∪ BLUE) = 12% or 0.12

Now, due to the fact that we have to choose ONE candy and only ONE candy at random, then they are mutually exclusive: Yes. Choosing a green and blue M&M is possible

b)P(yellow candy or red candy is;

P(YELLOW ∪ RED) = P(Y) + P(R)

P(YELLOW ∪ RED) = 20% + 23% = 43% or 0.43

Yes. Choosing a yellow and red M&M is possible

c) P(NOT PURPLE)

the probability of having a purple is;

P(PURPLE) = 22% or 0.22

So, the Probability of NOT having a PURPLE is 1 - 0.22 = 0.78

Will mark Brainliest! A stick has a length of $5$ units. The stick is then broken at two points, chosen at random. What is the probability that all three resulting pieces are longer than $1$ unit?

Answers

Answer:

0.16

Step-by-step explanation:

Length = 5 unitsNumber of broken sticks= 3Equal lengths =  5 units/3

See the picture attached for reference.

As you see the best points are the green areas which covers 2 out of 5 zones.

Since it is same for both broken points, the probability of  this is:

2/5*2/5 = 4/ 25 = 0.16

Answer is 0.16

The base of a triangle is 4 cm greater than the
height. The area is 30 cm. Find the height and
the length of the base
h
The height of the triangle is
The base of the triangle is​

Answers

Answer:

Step-by-step explanation:

Formula for area of a triangle:

Height x Base /2

Base (b) = h +4

Height = h

h + 4 x h /2 = 30cm

=> h +4 x h = 60

=> h+4h =60

=> 5h = 60

=> h = 12

Height = 12

Base = 12 +4 = 16

Write the equation in slope-intercept form, y = 4(x - 5) + 7x

Answers

The answer should be y = 11x-20

Answer:

y = 11x - 20

Step-by-step explanation:

y = 4(x - 5) + 7x

y = 4x - 20 + 7x

y = 11x - 20

In the game plan for model building on your multiple regression project, which of the following is true about the test that determines whether the interaction term(s) is/are significant in predicting y?
A. It will definitely be a t-test
B. It will either be a t-test or a best subset for partial) F-test depending on whether we kept or dropped the quadratic terms
C. It will definitely be a partial F-test
D. It will definitely be a global

Answers

Answer:

A. It will definitely be a t-test

Step-by-step explanation:

T-test is a type of inferential statistic test which used to determine whether there is a significant difference between mean of two group sets or variables.The t-test technique is widely used in hypothesis testing in statistics.  When the model building for game play is determine through regression analysis, it will require t-test to be conducted to reach a conclusion.

please help! 50 points!

Answers

Answer:

a) forming a bell

b) 5

c) 4.7

d) mean

is the correct answer

pls mark me as brainliest

please help me out! <3

Answers

Answer:

[tex]-1 \frac{3}{4}[/tex]

Step-by-step explanation:

Using this number line, we can plot our original number - [tex]\frac{3}{4}[/tex] (see picture attached)

Adding a negative is the same thing as subtracting - so we are subtracting [tex]2\frac{1}{2}[/tex] from  [tex]\frac{3}{4}[/tex].

To subtract this, we can break up [tex]2\frac{1}{2}[/tex]  into 3 parts: 1, 1, and [tex]\frac{1}{2}[/tex]. We can subtract each of these from the current number and see where we land up. (again see picture)

We land up at [tex]-1 \frac{3}{4}[/tex].

Hope this helped!

Scott start his banking account with 150 and is spending $7 per day on lunch . How would one describe the graph of this model?

Answers

Answer:

So this is giving us the slope the slope is y=-7x+150

Step-by-step explanation:

It is giving us the Y intercept which is $150  because thats how much he starts out with

It is giving us the slope -7 dollars because he is spending that everyday

Find the minimum sample size n needed to estimate for the given values of​ c, ​, and E. c​, ​, and E Assume that a preliminary sample has at least 30 members.

Answers

Answer:

hello your question is incomplete below is the complete question

Find the minimum sample size n needed to estimate μ For the given values of​ c, σ​, and E. c=0.98​, σ=6.5​, and E=22 Assume that a preliminary sample has at least 30 members.

Answer : 48

Step-by-step explanation:

Given data:

E = 2.2,

std ( σ ) = 6.5

c ( level of confidence ) = 0.98

To find the minimum sample size

we have to first obtain the value of  [tex]Z_{a/2}[/tex]  

note : a can be found using this relation :

( 1 - a ) = 0.98 ----- equation 1

a = 1 - 0.98 = 0.02

hence:  a/2 = 0.01

This means that P( Z ≤ z ) = 0.99  the value of z can be found using the table of standard normal distribution. from the table the value of z = 2.33

P( Z ≤ 2.33 ) = 0.99

To obtain the sample size n

[tex]n = (\frac{std*z}{E} )^{2}[/tex]

n = [tex](\frac{6.5*2.33}{2.2} )^2[/tex] =  (6.88409)^2

Therefore n ≈ 48

What is the average selling price
118,450
126,500
128,000
130,425
135,950

Answers

Answer:

A

Step-by-step explanat

What is nine thousandths as a decimal

Answers

Answer:

Nine thousandths = 0.009

Step-by-step explanation:

thousandths =  1/1000 = 0.001

nine thousandths = 9/1000 = 0.009

Answer:

.009

Step-by-step explanation:

9 thousandths as a decimal is 9/1000.  Which is the same 0.009

These figures are similar. The area of one is given. Find the area of the other. PLZ HELP

Answers

Answer: 6

Step-by-step explanation:

How many months does it take for $700 to double at simple interest of 14%?
• It will take
number.
months to double $700, at simple interest of 14%.

Answers

It will approximately take 7 months to double $700 at a %14 interest rate.
700•.14=98
98 divided into 700= 7.14

Find the volume of the cylinder. Round your answer to the nearest tenth.

Answers

Answer:

716.75 m^3

Step-by-step explanation:

Volume of a cylinder:

=> PI x R^2 x H

H = Height

R = Radius

=> PI x 3.9^2 x 15

=> PI x 15.21 x 15

=> PI x 228.15

=> 228.15 PI

           or

=> 228.15 x 3.14159

=> 716.75 m^3

In the xy -plane above, point C has coordinates (6,9).
Which of the following is an equation of the line that
contains points O and C

Answers

Y=3/2x that’s the answer
Other Questions
When a plant is receiving themaximum amount of lightintensity that it can withstand,what happens to the rate ofphotosynthesis?A. The rate levels off.B. The rate increases.C. The rate decreases. A mother, aged 60, wishes to withdraw monies from her variable annuity to pay for her son's college education. Which statement is true regarding the taxation of the withdrawal? A. The withdrawal is 100% taxable B. Any amount withdrawn above the cost basis is taxable C. Any amount withdrawn above the cost basis is taxable, and is subject to a 10% penalty tax D. The withdrawal is not subject to tax A customer has purchased 10,000 shares of Fromage stock, a Swiss cheese company. The stock is not traded in the United States. Fromage declares and pays a dividend of 15,000 Swiss Francs, which, when converted to dollars, equals $10,000. Switzerland imposes a 20% withholding tax on dividends repatriated outside its borders. How is the dividend reported on this investor's U.S. tax return A diffraction grating has 6000 lines per centimeter ruled on it. What is the angular separation (in degrees) between the second and the third orders on the same side of the central bright fringe when the grating is illuminated with a beam of light of wavelength 500 nm Archer receives a day's work of pay, p, for 5 days of mowing lawns. He spent half of his money on gas. Then he spent $5 on water. Now, he has $40 left. Which equation represents how much Archer would get paid each day of mowing lawns? Q-21: The regions of inequalities are also called: A) Planes B) Lines C) Half planes D) None of these seigel co. maintains a defined-benefit pension plan for its employees. at each balance sheet date, seigel should report a pension asset / liability equal to the 28.Neethi had 814cups of flour and 3 liters of juice with her. She decided to make cupcakes anddistribute juice for her birthday.a) A cupcake requires 34cup of flour. How many cupcakes can she make? Carlos wants to purchase a new computer and go to the Caribbean for spring break. The computer is priced at $1,299, and the vacation is priced at $750. He has only $1,537 in his checking account, so he cannot afford to purchase both. After much thought, Carlos buys the computer and writes a check for $1,299. Identify what role mo Calculate the energy required to heat of 1.50 kg silver from -7.8 C to 15.0 C . Assume the specific heat capacity of silver under these conditions is .0235 J*g^-1*K^-1 . Be sure your answer has the correct number of significant digits. Why are G proteins found in cell membrane of the cell ? A coin is placed at a depth of 15 cm in a beaker from the surface of water. Therefractive index of water is 4/3.Calculate height through which the image of the coin is raised. Look at the text below . If theres is an error with subject-verb agreement, select the incorrect verb and type it correctly. Otherwise, submit the text without making any changes.1.Neither toxic pollutants nor loud engine noise are a problem 2.for food cart operators who use curbside charging stations instead 3.of gasoline-powered generatiors Based on these descriptions, which man is impotent? Jacob has a low sperm count. Reuben has a blocked vas deferens. Leon is unable to sustain an erection. Troys testes have not descended into the scrotum. An HCl solution has a concentration of 0.09714 M. Then 10.00 mL of this solution was then diluted to 250.00 mL in a volumetric flask. The diluted solution was then used to titrate 250.0 mL of a saturated AgOH solution using methyl orange indicator to reach the endpoint.Required:a. What is the concentration of the diluted HCI solution? b. If 7.93 mL of the diluted HCI solution was required to reach the endpoint, what is the concentration of OH- in solution? c. What is the concentration of Ag+ in solution?d. What is the Ksp expression for the dissolution of AgOH? PLEASE HELP ME!!!!!!!(I have an idea of what the answer is but I just want to make sure) Read and choose the option that answers the question. Hola, amigos! Me llamo Julieta y tengo quince aos. En las noches me bao a las ocho de la noche. Me lavo el pelo y me seco el pelo los domingos, martes y jueves. Despus, me pongo el pijama y me arreglo antes de salir del bao. Me duermo a las nueve y media de la noche. Based on the reading, select the "yo-go" verb used in the paragraph. Wash Bathe Fix Have (I think that this is the answer) Considere que a empresa vale dos lirios Ltda, recebeu por meio de caixa, o valor registrado an conta de clientes referente a venda efetuada no ms de marco/2020- no valor de $3.400,00. Qual a alternativa que corresponde o mtodo das partidas dobradas? An octagonal pyramid ... how many faces are there, how many vertices and how many edges? A triangular prism ... how many faces are there, how many vertices and how many edges? a triangular pyramid ... how many faces are there, how many vertices and how many edges? Which of these statements is false Which primary source document could help a historian understand why the US Civil War started?